- Wed Mar 30, 2016 6:00 pm
#22632
Question #21: Strengthen. The correct answer choice is (B).
The stimulus describes a new gadget used to keep cattle in their pastures. The device uses GPS tracking and makes noise in the cow’s ears whenever the cow wanders off its pasture, steering it back home. Needless to say, coolness does not come cheap: apparently, outfitting all the cows with this device would be a far more expensive solution than, say, building fences to keep them contained. Nevertheless, the manufacturer is confident that the ranchers will shell out the cash anyway, and the stem asks us to support this prediction.
While technically a Strengthen question, it would be just as useful to think of it as a Resolve question: why would the ranchers purchase the device at its current price if there are cheaper ways of keeping the cattle in check? Maybe they are loaded and willing to splurge on cool gadgets? Possible, but highly unlikely. The LSAT assumes, for the most part, that people are rational actors. The correct answer choice must establish why purchasing the device would be a rational choice, not one driven by profligacy or stupidity.
Answer choice (A): At first glance, this may seem like an attractive answer. Recall, however, that you are supposed to strengthen the manufacturer’s prediction as it is: ranchers will purchase the device at its current price.
Answer choice (B): This is the correct answer choice, suggesting that ranchers will not need to outfit every single cow with the device. If only herd leaders get the gadget, the overall cost of using the device could be reduced to levels at or below the cost of building fences. Purchasing the device at its current price could be a rational choice after all, strengthening the prediction made by the device’s maker.
Answer choice (C): Just because the device causes no harm does not mean that the ranchers will necessarily buy it at its current price. They might, but only if the alternative means of keeping cattle in their pastures are somehow harmful. We do not know if they are, which is why answer choice (C) does not necessarily present a comparative advantage.
Answer choice (D): This is the Opposite answer, as it makes the manufacturer’s prediction even more baffling: if the device is just as effective as fences at keeping cattle in their pastures, then why not use the cheaper solution instead?
Answer choice (E): As with answer choice (A), this one does not support the exact prediction made in the stimulus. The manufacturer believes that ranchers will purchase the device at its current price, not at a discount.
The stimulus describes a new gadget used to keep cattle in their pastures. The device uses GPS tracking and makes noise in the cow’s ears whenever the cow wanders off its pasture, steering it back home. Needless to say, coolness does not come cheap: apparently, outfitting all the cows with this device would be a far more expensive solution than, say, building fences to keep them contained. Nevertheless, the manufacturer is confident that the ranchers will shell out the cash anyway, and the stem asks us to support this prediction.
While technically a Strengthen question, it would be just as useful to think of it as a Resolve question: why would the ranchers purchase the device at its current price if there are cheaper ways of keeping the cattle in check? Maybe they are loaded and willing to splurge on cool gadgets? Possible, but highly unlikely. The LSAT assumes, for the most part, that people are rational actors. The correct answer choice must establish why purchasing the device would be a rational choice, not one driven by profligacy or stupidity.
Answer choice (A): At first glance, this may seem like an attractive answer. Recall, however, that you are supposed to strengthen the manufacturer’s prediction as it is: ranchers will purchase the device at its current price.
Answer choice (B): This is the correct answer choice, suggesting that ranchers will not need to outfit every single cow with the device. If only herd leaders get the gadget, the overall cost of using the device could be reduced to levels at or below the cost of building fences. Purchasing the device at its current price could be a rational choice after all, strengthening the prediction made by the device’s maker.
Answer choice (C): Just because the device causes no harm does not mean that the ranchers will necessarily buy it at its current price. They might, but only if the alternative means of keeping cattle in their pastures are somehow harmful. We do not know if they are, which is why answer choice (C) does not necessarily present a comparative advantage.
Answer choice (D): This is the Opposite answer, as it makes the manufacturer’s prediction even more baffling: if the device is just as effective as fences at keeping cattle in their pastures, then why not use the cheaper solution instead?
Answer choice (E): As with answer choice (A), this one does not support the exact prediction made in the stimulus. The manufacturer believes that ranchers will purchase the device at its current price, not at a discount.